Log Derivative of a Function: Find Derivatives of f(c,l) with Example"

  • Thread starter Thread starter beaf123
  • Start date Start date
  • Tags Tags
    Derivative Log
Click For Summary
The discussion centers on finding the derivatives of the function f(c,l) = log(c - ψ(1-l)θ) with respect to the variables l and c. The correct derivative with respect to c is 1/(c - ψ(1-l)θ), while the derivative with respect to l involves applying the chain rule, resulting in θψ(1-l)θ-1/(c - ψ(1-l)θ). Participants emphasize the importance of proper parentheses in mathematical expressions to avoid errors. Missteps in notation can lead to incorrect results, which may affect grading. Overall, clarity in mathematical writing is crucial for accurate problem-solving.
beaf123
Messages
40
Reaction score
0

Homework Statement



f(c,l) = log(c - ψ(1-l)^θ )

What is the derivative of this function wrt. l and c?

Homework Equations



I know that the derivative of log (x) = 1/x

The Attempt at a Solution



I got wrt c:

1/ c - ψ(1-l)θ

and wrt l: θψ(1-l)^θ-1 / c - ψ(1-l)^θ
 
Physics news on Phys.org
Remember the chain rule.
##\frac{d}{dt} f(g(t)) =f'(g(t)) * g'(t). ##
If log is your f, and the stuff inside is your g, your first part, f'(g) would be ##\frac{1}{c-\psi (1-l)^\theta}##. Then you will need to multiply by the derivative of g wrt. whichever variable you are looking at.

edit: This may very well be what you did, but without proper parentheses, I can't tell.
 
Thak you. Yes, that is wahrt I did.
 
beaf123 said:

Homework Statement



f(c,l) = log(c - ψ(1-l)^θ )

What is the derivative of this function wrt. l and c?

Homework Equations



I know that the derivative of log (x) = 1/x

The Attempt at a Solution



I got wrt c:

1/ c - ψ(1-l)θ

and wrt l: θψ(1-l)^θ-1 / c - ψ(1-l)^θ

Those are wrong: you should not be getting
f_c(c,l) = \frac{1}{c} - \psi(1-l) \theta
which is exactly what you wrote.
 
Yeah. I messed up the paranthesis. Thanks for telling me what I should not be getting though
 
beaf123 said:
Yeah. I messed up the paranthesis. Thanks for telling me what I should not be getting though

Well, maybe with proper parentheses, and fixing up ##(1-l)^{\theta}##, your result could be correct. It would be a shame to lose marks on an assignment by not using parentheses when it really takes little extra time.
 
Question: A clock's minute hand has length 4 and its hour hand has length 3. What is the distance between the tips at the moment when it is increasing most rapidly?(Putnam Exam Question) Answer: Making assumption that both the hands moves at constant angular velocities, the answer is ## \sqrt{7} .## But don't you think this assumption is somewhat doubtful and wrong?

Similar threads

  • · Replies 4 ·
Replies
4
Views
1K
  • · Replies 8 ·
Replies
8
Views
2K
  • · Replies 2 ·
Replies
2
Views
2K
  • · Replies 1 ·
Replies
1
Views
1K
  • · Replies 2 ·
Replies
2
Views
2K
  • · Replies 12 ·
Replies
12
Views
2K
  • · Replies 8 ·
Replies
8
Views
1K
  • · Replies 2 ·
Replies
2
Views
1K
  • · Replies 8 ·
Replies
8
Views
2K
  • · Replies 11 ·
Replies
11
Views
2K